Вы находитесь на странице: 1из 51

Chapter 8 Responsibility Accounting 60

MULTIPLE CHOICE

Basic Concepts
1. Which of these assertions refer to responsibility accounting?
1. Costs and revenues are identified with individuals for better control and
performance appraisal.
2. Performance reports under this concept include variance of actual amounts
versus plan.
3. Third parties who are external users are the main recipients of information.
4. Only expenses which are directly under the control of managers should ideally be
charged to them.
A. Assertions 1, 2 and 4 only.
B. Assertions 1 and 4 only.
C. Assertions 1 and 2 only.
D. All four assertions. (rpcpa)

1. A
? The assertions that refer to responsibility accounting.
 Statements 1, 2, and 4 refer to responsibility accounting (choice-letter “a” is correct)
Responsibility accounting divides the entire organization into responsibility
centers where each center has a manager who has the authority, responsibility, and
accountability over the incurrence or non-incurrence of transactions. The
responsibility centers are classified as cost centers, revenue centers, profit centers,
and investment centers. Cost center managers have control or influence over the
non- incurrence or non-incurrence of costs and is evaluated through cost variance
analysis. Revenue center managers have control over the process of generating
revenues and are evaluated through the revenue variance analysis. Profit center
managers have control both for revenue and costs and are assessed through the
gross profit variation analysis and contribution margin or controllable margin analysis.
Investment center managers control the investment decisions to be made and are
normally appraised using the return on investment (ROI) model, and the residual
income model.
The areas of authority and responsibility are defined for better control and
performance appraisal (i. e., accountability). Performance reports and appraisals
include variances between actual an planned data. Only controllable revenues, costs,
and investments are to be evaluated as to the overall effectiveness and efficiency of
the manager. The main recipient or user of the information generated in the
responsibility accounting system is the management, and not external parties
(assertion no. 3 is wrong).

2. The basic purpose of a responsibility accounting system is


A. Budgeting. C. Authority
B. Motivation. D. Variance analysis. (cma)

2. B
? The basic purpose of responsibility accounting system.
 Responsibility accounting is designed to complement a decentralized model of
managing where authorities are devolved to the level of operating people who have
Chapter 8 Responsibility Accounting 61

the training, qualification and sense of responsibility to use the delegated authority in
making decisions that are expected to produce hands-on, quick and more customer-
oriented decisions. The essence of decentralization (i.e., responsibility accounting) is
to highlight understanding, involvement, commitment, and ownership of particular
process or undertaking with the end-in-view of attaining its desired results. This
great shift in managing people is designed to uplift motivation and higher productivity.
Choice-letters “a” and “d” are incorrect because budgeting and variance analysis
are techniques applied to carry the purposes of responsibility accounting system,
their advantages are secondary to the basic purpose of motivation resulting to
increased efficiency and productivity. Choice-letter “c” is incorrect because
responsibility accounting is not designed to create authority but as a means of
distributing (i.e., delegating) authority with the condition that such authority and use of
authority shall be evaluated at a later date.

3. Responsibility accounting:
A. Is the most formal communication device within an enterprise
B. Encourages managers and other employees to achieve enterprise goals,
not just their own individuals goals
C. Encourages managers to focus on a single issue of evaluation
D. Deals with the reporting of information to facilitate control of operations
and evaluation of performance
(rpcpa)

3. B
? The best description about responsibility accounting.
 The purpose of responsibility accounting is to motivate management divisions in
adherence to overall company objectives. This concept is called “goal congruence.”
Choice-letter “a”, ”c” and “d” are statements that indirectly describe the effects of
responsibility accounting but are not major reasons in establishing a responsibility
accounting system in the business.

4. That kind of accounting concerned with providing information to management in


making decisions about the operations of the business
A. Responsibility accounting. C. Management accounting.
B. Cost accounting. D. Full cost accounting. (rpcpa)

4. C
? The kind of accounting concerned with providing information to management in
making decisions about the operations of the business.
 Choice-letter “c”, management accounting, is the correct answer. Management
accounting provides information to management in making business decisions about
the operations of the business.
Choice-letter “a” is not the best answer because responsibility accounting is only
one of the techniques applied in management accounting to provide information to
management. Choice letter “b, cost accounting, is incorrect because it focuses only
to accounting for the cost of production. Choice-letter “d”, full accounting, is only one
of the techniques used in management accounting to generate reports and
information.
Chapter 8 Responsibility Accounting 62

5. A successful responsibility accounting reporting system is dependent upon


A. The correct allocation of controllable variable costs.
B. Identification of the management level at which all costs are controllable.
C. The proper delegation of responsibility and authority.
D. A reasonable separation of costs into their fixed and variable components since fixed
costs are not controllable and must be eliminated from the responsibility report. (cma)

5. C
? A condition that results to a successful responsibility accounting reporting system.
 Responsibility accounting promotes decentralization of authority and evaluation of
performance based on the concept of controllability. Hence, the success of a
responsibility accounting system depends on well-defined lines of authority and
responsibility.
Choice-letter “a” is incorrect because controllable costs are defined within the
sphere of managerial authority, direct and identifiable, and not allocated. Choice-
letter “b” is incorrect because responsibility accounting does not only limit on the
identification of management levels at which costs are controllable but covers actual
implementation and evaluation of actual performance. Choice-letter “d” is incorrect
because there are fixed costs that are direct and controllable within the ambit of
one’s managerial authority and such are included in the responsibility report.

6. In responsibility accounting, there are two (2) types of reports distinguished as to goals
or objectives
A. Trend analysis reporting and comparative reporting.
B. Responsibility performances reporting and information reporting.
C. Operations reporting and financial condition reporting.
D. Horizontal reporting and vertical reporting. (rpcpa)

6. B
? Two types of reports distinguished as to goals or objectives.
 Choice-letters “a”, “c”, and “d” are financial statements analysis report (e.g., trend
analysis, comparative analysis, vertical analysis, horizontal analysis income
statement analysis, and balance sheet analysis). Their goal is to analyze and
interpret the performance in the previous period compared with the defined standards
set by the organization.
Choice-letter “b” is the correct answer. Responsibility performance reporting
emphasizes the performance of the manager and his division over items of which he
has control, together with the corresponding variance analysis. Information reporting
emphasizes the generation of relevant information to decision makers without the
corresponding intricate and detailed analysis explaining the process and outcome of
the provided information.

7. When used for performance evaluation, the generated reports in a responsibility


accounting system should
A. Not be related to the organization structure.
B. Not include variances between actual results and budgeted amounts of
controllable costs.
Chapter 8 Responsibility Accounting 63

C. Not distinguish between controllable and uncontrollable costs.


D. Not include allocated fixed manufacturing overhead. (rpcpa)

7. D
? A correct description about the generated reports in a responsibility accounting
system.
 Choice-letters “a”, “b”, and “c” are false statements with regard to performance
evaluation in a responsibility accounting environment. The responsibility accounting
report must be in accordance with organizational structure of the firm. It should
separate the controllable from the non-controllable costs to define the areas of
authority, responsibility and accountability. It should include variances of actual from
the plans or standards. It should define the segment manager’s controllable costs
from that of the segment’s direct margin itself. A segment manager should be
evaluated on controllable margin and not on segment’s income. A controllable
margin does not include an allocated fixed overhead. Choice-letter “d” is correct).

8. Which of these are among the qualities of a good report under the concept of
responsibility accounting?
1. It should be consistent in form and content for each issue.
2. It should be prompt, timely and regularly issued.
3. It should easily be understood by users as to the contents, their
significance and how to use them.
4. It should be able to pinpoint who is to blame as a pre-requisites to
explain variances.
5. It should highlight efficiencies and inefficiencies.
6. It should be comparative and analytical.
7. It should be comprehensive as to include all details that can possibly be
contained in the report.
A. All except 4 and 7.
B. All seven statements.
C. All except 4, 5 and 6.
D. Statements 1, 2, 3, 4 and 7 only. (rpcpa)

8. A
? The qualities of a good report under the responsibility accounting concept
 A responsibility accounting report should have the following characteristics:
consistency, timeliness, understandability, defined areas of authority, responsibility
and accountability, performance measurement, comparative and analytical, and
adequacy in coverage. Additionally, a good report must be user-friendly, output-
oriented, specific, and time-bound.
Item no. 4 is irrelevant because a good responsibility accounting report
automatically identifies who is accountable for the variances and that is no other than
the person who has a control over the incurrence and non-incurrence of transaction.
Item no. 7 is also irrelevant because a good responsibility accounting report should
be comprehensive to include important areas for consideration but not too detailed
that may make the more important data buried in a sea of superfluous specifics.
Hence, the correct answer is choice-letter “a”.
Chapter 8 Responsibility Accounting 64

9. The CEO of a rapidly growing high-technology firm has exercised centralized


authority over all corporate functions. Because the company now operates in four
states, the CEO is considering the advisability of decentralizing operational control
over production and sales. Which of the following conditions probably will result
from and be a valid reason for decentralizing?
A. Greater local control over compliance with federal regulations.
B. More efficient use of headquarters staff officials and specialist.
C. Quicker and better operating decisions.
D. Greater economies in purchasing. (cia)

9. C
? A condition that would probably result from and be a valid reason for decentralizing.
 Decentralization results to delegation of authority among operating managers who
have the technical skill and competent understanding of the conditions that
characterize the operating activities. This setup is expected to produce quicker and
better operating decisions, thereby eliminating delays and increasing efficiency and
productivity.
Choice-letter “a” is incorrect because compliance with government regulations
which are normally mandatory or prescriptive and not a matter of controls which are
internal within the organization. Choice-letter “b” is incorrect because
decentralization diminishes the workload of headquarters staff officials and specialist
since responsibilities are now transferred from the central office to the created
organizational independent units. Choice-letter “d” is incorrect because
decentralization will make purchasing now a concern and within the ambit of power of
operating managers who are tasked to make the entire process under his authority
more efficient and functional.

10. Which one of the following will not occur in an organization that gives managers
throughout the organizations maximum freedom to make decisions.
A. Individual managers regard the managers of other segments as if they are
transacting with external parties.
B. Two divisions of the organization having competing models that aim for the same
market segments.
C. Delays in securing approval for the introduction of new products.
D. Greater knowledge of the marketplace and improved service to customers. (cma)

10. C
? A situation that will not occur in a decentralized organization.
 In a decentralization organization, individual managers are independent with one
another and treat the manager of other segments as they do external parties, two or
more managers may be competing in the same market, and organizational segments
have greater knowledge of the marketplace and services to customers are expected
to be more improved. Because of decentralization, delays in securing approval to
matters that concern segment operations would be most likely avoided.
Choice-letters “a”, “b”, and “d” are incorrect because they are expected to be
recurring in a decentralized operations.
Chapter 8 Responsibility Accounting 65

Choice-letter “c” is correct because in a decentralized organization operating


mangers are empowered to make decisions within the sphere of authority given to
them. This set up eliminates or minimizes delays in making decisions.

11. Which of the following is most likely to be a disadvantage of decentralization?


A. Lower-level employees will develop less rapidly than in a centralized
organization.
B. Lower-level employees will complain of not having enough to do.
C. Top management will have less time available to devote to unique problems.
D. Lower-level managers may make conflicting decisions. (cia)

11. D
? A most likely disadvantage of decentralization.
 Decentralization is the distribution of organizational authority among trusted and
qualified personnel who are empowered to make decisions within the sphere of their
authority. This strategy would create operationally independent managers whose
performance shall be evaluated later based on defined and agreed standards. As a
consequence, operating processes are expected to be more customer-oriented,
quicker in making decisions, and highly competitive. On the other flip of analysis,
decentralization may result to conflicting decisions of operating managers that would
derail overall goal congruence and may lead instead to suboptimization where the
interest of an independent unit may be upheld against that of the overall interest of
the organization.
Choice-letter “a” is incorrect because decentralization leads to quicker
development of lower-level managers. Choice-letter “b” is incorrect because lower-
level managers are held responsible and as such shall be in control of their activities
and schedule of activities to get their job done. Choice-letter “c” is incorrect because
decentralization will reduce operating workload of top management who has more
time now in developing and evaluating strategic decisions.

12. To make goal setting effective and worthwhile, the goals should be
A. Just beyond what subordinates are likely to reach.
B. Quantitative and approximate.
C. Based on superior performer’s output.
D. Specific, objective, and verifiable. (cia)

12. D
? The characteristics needed to make goal setting effective and worthwhile.
 Goals must be SMART (i.e., specific, measurable, attainable, realistic, and time-
bounded) to be effective. The nearest description among the choices is choice-letter
“d”.
Choice-letter “a” is incorrect because setting goals beyond what subordinates are
likely to reach does not ensure effective performance but may, in some instances,
demotivate personnel to perform at their best. Choice-letter “b” is incorrect because
goals are supposed to be realistic and not based on mere approximations. Choice-
letter “c” is also incorrect because goals are supposed to be attainable to be
effective.
Chapter 8 Responsibility Accounting 66

13. Making segments disclosures is an advantages to a company because it


A. Facilitates evaluation of company management by providing data on particular
segments
B. Eliminates the interdependence of segments
C. Masks the effect of intersegment transfers.
D. Provides competitors with comparative information on the company’s
performance. (cia)

13. A
? An advantage of making segment disclosures.
 Segment disclosures provide results of activities of specific segments. These
disclosures are of value to management in evaluating the parts (e.g., segments) of
the whole organization. This information is normally used for management purposes
only and not for general use because such information may be irrelevant to the
interest of other parties in business. This advantage of segment disclosure is
inherently internal in organizational reporting.
Choice-letter “b” is incorrect because segment reporting does not eliminate
segment interdependence with respect to operations and alliances. Choice-letter c”
is incorrect because segment disclosures even unveil specific performance of
organizational segments (or business units). Choice-letter “d” is incorrect because it
is of a disadvantage than an advantage to the reporting company.

14. Costs are accumulated by a responsibility center for control purposes when using
Job-Order Costing Process Costing
A. Yes Yes
B. Yes No
C. No No
D. No Yes (aicpa)

14. A
? The costing method(s) used in accumulating costs by responsibility centers.
 Costing methods follow organizational design and structures. Job-order costing,
process costing, activity-based costing, life-cycle costing, and other costing methods
may be used in a responsibility reposting system, where the emphasis is on the
delegation of authority and responsibility that sets the framework for the costing
system to be used. Besides, responsibility accounting is a managerial strategy to
accumulate reports and the costing methods are techniques embodied in such
strategy.

Controllability
15. In deciding how or which costs should be assigned to a responsibility center is the
degree of
A. Avoidability. C. Controllability
B. Variability. D. Relation to department.

1.5. C
? The basis of deciding which costs should be assigned to a responsibility center.
Chapter 8 Responsibility Accounting 67

 The premise of responsibility accounting is the allowance for freedom to make


decisions. And each responsibility manager should be evaluated on the scope of
authority or controllability entrusted to him. Once an item, object, process, etc. is
within the ambit of a manager’s authority, then that item, object, process, etc. is
controllable with respect to such manager. Choice-letter “c” is the correct answer.
Choice-letter “a” is incorrect because avoidability is not the only measure in
determining the degree of responsibility, non-avoidability could also be a measure.
Choice-letter “b”, variability, is not only a measure because non-variability may also
be included as a degree of assigning costs to a responsibility center manager.
Choice-letter “d” is incorrect because a cost relation to the department is not the
basis of evaluating the performance of a manager but rather on the costs that he
controls in his department.

16. In a responsibility accounting a center’s performance is measured by controllable


costs. Controllable costs are best described as including.
A. Differential costs.
B. Only those costs that the manager can influence in the current time period.
C. Incremental and fixed costs.
D. Only discretionary cost. (rpcpa)

16. B
? The best description of items included in the controllable costs.
 Controllable costs are those which incurrence depends on the power, authority,
discretion, or influence of a manager. If a manager does not have the power to
decide on the incurrence or non-incurrence of a cost, such cost is non-controllable as
far as that a manager is concerned (choice-letter ‘b” is correct). The controllability or
non-controllability of a cost depends on where does the manager situate in the
organizational hierarchy and the scope and platitude of power (or authority) he has in
the organization.
Choice-letter “a” and “d” are incorrect because not all differential costs and
discretionary costs are always controllable costs. Choice-letter “c” is incorrect
because fixed costs are normally not controllable by an operating manager.

17. Controllable costs for responsibility accounting purposes are those costs that are
directly influenced by
A. A given manager within a given period of time.
B. A change in activity.
C. Production volume.
D. Sales volume (aicpa)

17. A
? The one that influences a controllable cost.
 Controllable costs are those which incurrence or non-incurrence is decided upon or
influenced by a manager, choice-letter “a” is the right answer.
Choice-letters “b”, “c”, and “d” are incorrect because controllable costs are not
influenced by a change in activity, production volume and sales volume.

18. Among the management accounting concepts is controllability which means


Chapter 8 Responsibility Accounting 68

A. It is necessary at all times to identify the responsibilities and key result areas of
the individuals within the organization.
B. Management accounting must ensure that flexibility is maintained in assembling
and interpreting information.
C. Management accounting identifies elements or activities which management can
or cannot influence, and seeks to arrest risk and sensitivity factors.
D. Accounting information must be of such quality that confidence cab be placed in it. (rpcpa)

18. C
? A true statement about the concept of controllability.
 Controllability refers to the ability of a person to decide upon or influence the
incurrence or non-incurrence of an item or activity. The degree of one’s control over
an item or activity depends on the nature, strength, and amount of authority given to
him to minimize risks and sensitivity factors (choice-letter “c” is correct).
Choice-letter “a” is incorrect because it relates to the concept of proper
delegation. Choice-letter “b” is also incorrect because it pertains to flexibility.
Choice-letter “d” is incorrect because it speaks of confidentiality.

19. The process of attributing proportion of items of costs among cost centers is called
A. Overhead absorption. C. Cost apportionment.
B. Absorption of indirect cost. D. Cost allocation. (rpcpa)

19. D
? The process of attributing proportion of items of costs among cost centers.
 Choice-letter “d” is the correct answer. Cost allocation refers to the process of
distributing common costs among costs centers that are benefited by the incurrence
of such costs. Choice-letter “a” is incorrect because overhead absorption relates to a
costing method where total costs incurred in the production process, whether variable
or fixed, are included as product costs. Choice-letter “b” is also incorrect because it
is the same as choice-letter “a”, only phrased differently. Choice-letter “c” is incorrect
because cost apportionment implies distribution of costs to cost centers without
establishing whether a center is benefited by the incurrence of such costs or not.

20. Managers are most likely to accept allocations of common costs based on
A. Cause and effect. C. Fairness.
B. Ability to bear. D. Benefits received. (gleim)

20. A
? The most acceptable basis of allocating common costs.
 Common costs or indirect costs are those incurred to carry an activity without clearly
or directly identifying which segment of the activity is the cost related with. These
costs are to be distributed among department segments (or departments) of the
entire business process. Allocating common costs should be approach with caution
for this may lead to demotivational atmosphere. Essentially, common costs should
be allocated according to relationship. Or otherwise stated, according to factors that
induce them to be incurred. This means identifying activity drivers that induce activity
costs, or simply based on cause-and-effect relationship.
Chapter 8 Responsibility Accounting 69

Choice-letter “b” is incorrect because ability to bear would unduly burden


segments that are more capable even if not benefited from the incurrence of such
costs. Choice-letter “c” is incorrect because fairness is a relative measure that
would have to be established and if ever established may not even yield motivational
results to others who have a different interpretation of what is fair and what is not.
Choice-letter “d” is incorrect because cause-and-effect should be exhausted first, and
if not applicable, the rationale of benefits received may be used. In short, cause-and-
effect is a more prioritized model of allocating common costs than the benefit-
received criterion.

21. Which one of the following firms is likely to experience dysfunctional motivation on the
part of its managers due to its allocation methods?
A. To allocate depreciation of forklifts used by workers at its central warehouse,
Shahlimar Electronics uses predetermined amounts calculated on the basis of
the long-term average use of the services provided.
B. Manhattan Electronics uses the sales revenue of its various divisions to allocate
costs connected with the upkeep of its headquarters building. It also uses ROI to
evaluate the divisional performances.
C. Rainier Industrial does not allow its service departments to pass on their cost
overruns to the production departments
D. Tashkent Auto’s MIS is operated out of headquarter and serve its various
divisions. Tashkent’s allocation of the MIS- related costs to its divisions is limited
to costs the divisions will incur if they were to outsource their MIS needs.(cma)

21. B
? A firm that is likely to experience dysfunctional motivation on the part of its managers
due to allocation methods.
 Costs allocation may be also based on benefit-cost criterion, on the rate of benefit
received arising from a cost or activity. Choice-letter “b” will likely result to
dysfunctional motivation since allocating up-keeping costs of headquarter building
based on sales revenue would unduly allocate great amount of costs to sales
department and none to non-revenue producing departments which may be
occupying more building space.
Choice-letters “a”, “c”, and “d” are incorrect because they use allocation methods
based on the ratio of benefit derived out of the activity costs, and therefore are not,
and would not result, to dysfunctional motivation.

Costs centers
22. What is the name given to a unit or a function of an organization that is headed by a
manager who has direct responsibility for its performance?
A. Responsibility center. C. Business entity.
B. Cost unit. D. Budget center. (rpcpa)

22. A
? A name given to a unit headed by a manager who has direct responsibility for its
performance.
 Choice-letter “a” is correct. A responsibility center is an organizational segment
which performance is under the control and accountability of a responsible manager.
Chapter 8 Responsibility Accounting 70

As such, the manager should be evaluated on the degree of authority and


responsibility he has accepted.
Choice-letter “b” is incorrect because a cost unit is only one of those
responsibility centers normally established within an organization. The other
responsibility centers are the revenue center, profit center, and investment center.
Choice-letter “c” is incorrect because a business entity implies independence from
other entities that the business has interest of. A business entity may also be
understood as an investment center that is only one of the responsibility centers
identified in an organization. Choice-letter “d”, budget center, is a term used within
the operations of budgeting system that is only one of the techniques applied in an
organization structured within the principles of responsibility accounting.

23. Cost centers are


A. Units of product or service for which costs are ascertained.
B. Amounts of expenditure attributable to various activities.
C. Function or locations for which costs are ascertained for control purposes.
D. A section of an organization for which budgets are prepared and control
exercised. (rpcpa)

23. D
? A description of a cost center.
 A cost center is a segment in an organization where costs are budgeted and
controlled. Choice-letter “d” is correct. Choice-letter “a” is incorrect because it refers
to the units of product or service and not to an organizational segment. Choice-letter
“b’ is incorrect because it refers to activities and not organizational segments.
Choice-letter “c” is also incorrect because it refers only to location or functions for
which costs are ascertained for control purposes, without relevant mention to
reporting and budgeting purposes.

24. The least complex segments or area of responsibility for which costs are all allocated
is a (n)
A. Profit center. C. Contribution center.
B. Investments center. D. Cost center. (cma)

24. D
? The least complex segment or area of responsibility for which are costs are allocated.
 Cost center mangers have the authority to control the incurrence or non-incurrence of
costs, and as such cost centers are the least complex segments for which costs are
allocated.
Choice-letter “a” is incorrect because profit center controls both costs and
revenues and is more complex than a cost center. Choice-letter “b” is incorrect
because investment center controls costs, revenues, profit and investments and is
the most complex responsibility center. Choice-letter “c” is incorrect because
contribution center may also be referred to as profit costs.

25. Which of the following techniques would be best for evaluating the management
performance of a department that is operated as cost center?
Chapter 8 Responsibility Accounting 71

A. Return-on-assets ratio. C. Payback method.


B. Return-on-investment ratio. D. Variance analysis. (cia)

25. D
? A technique of evaluating a cost center management performance.
 Cost center managers are to be evaluated on the span of authority delegated to
them. Their authority only includes control of costs as to their incurrence or non-
incurrence. Actual costs should be compared with the planned (or standard) costs to
identify variations or differences. This technique of evaluating cost center managers
is called as cost variance analysis.
Choice-letters “a” and “b” are incorrect return on assets ratio is used to evaluate
the performance of investment center managers. Choice-letter “c”, return on
investment ratio, is incorrect because payback period is used in evaluating capital
investment opportunities.

26. If a manufacturing company uses responsibility accounting, which one of the following
items is least likely to appear in a performance report for a managers of an assembly
line?
A. Supervisory salaries. C. Repairs and maintenance.
B. Materials. D. Depreciation on equipment. (cma)

26. D
? An item not likely to appear in a performance of a manager in an assembly line.
 The performance report of an assembly line manager shall highlight only those costs
that he has control over with. Depreciation of equipment is uncontrollable to the
assembly line manager because he has no authority to decide on what equipment to
buy. Traditionally, the decision on what equipment to buy is delegated to the
investment center manager who, therefore, has the authority to control depreciation
of equipment.
Choice-letters “a”, “b”, and “c” are incorrect because they are all controllable with
respect to the assembly line manager.

27. Which of the following items of cost would be least likely to appear in a performance
report based on responsibility accounting techniques for the supervisor of an
assembly line in a large manufacturing situation?
A. Materials. C. Repairs and maintenance.
B. Supervisor’s salary. D. Direct labor. (rpcpa)

27. B
? The cost that would least likely appear in a performance report of an assembley line
supervisor.
 A cost that would least likely appear in a performance report of an assembly
supervisor based on responsibility accounting includes those that are non-
controllable by the supervisor. A cost is controllable as to the supervisor if he has the
power to decide or influence the incurrence or non-incurrence of costs. Examples of
such cost are direct materials, direct labor, repairs and maintenance, supplies,
electricity, water, and security.
The supervisor’s salary is controlled by his superior, hence, choice-letter “b” is
the correct answer.
Chapter 8 Responsibility Accounting 72

28. The receipts of raw materials used in the manufacture of products and the shipping of
finished goods to customers is under the control of the warehouse supervisor, whose
time is spent approximately 60% on receiving and 40% on shipping activities.
Separate staffs for these operations are employed. The labor-related costs for the
warehousing function are as follows:
Warehouse supervisor’s salary P 40,000
Receiving clerk’s wages 75,000
Shipping clerk’s wages 55,000
Employee benefit costs (30% of wage and salary costs) P 51,000
Total P221,000
The company employs a responsibility accounting system for performance-reporting
purposes. Costs are classified as period or product costs. What is the total of labor-
related costs reported as product costs under the control of warehouse supervisor?
A. P 97,500 C. P130,000
B. P128,700 D. P221,000 (cia)

28. A
? The total of labor-related costs reported as product costs under the control of
warehouse supervisor.
 The costs to be included are not only product costs, but are also controllable costs to
warehouse supervisor. The warehouse supervisor’s salary, although a product costs,
is not controllable to the warehouse supervisor and should not be included in the list.
The shipping costs are not product costs, although controllable, and are not to be
included in the required list. This leaves only the receiving clerk’s wages and its
corresponding employee benefit costs as product costs and controllable costs with
respect to the warehouse supervisor. The total labor-related costs reported as
product costs under the control of warehouse supervisor is P97,500 (i.e., P75,000 x
130%).

Profit centers
29. Responsibility accounting defines an operating center that is responsible for revenue
and costs as a (n)
A. Profit center. C. Division.
B. Revenue center. D. Operating unit. (cma)

29. A
? A responsibility center that is responsible for revenue and costs.
 Profit centers are responsible for the generation of revenues and incurrence of costs.
Choice-letter “b” is incorrect because revenue center managers are responsible
only for the creation of revenue. Choice-letter “c” is incorrect because a division may
be a cost center, a revenue center, profit center, or even an investment center.
Choice-letter “d” is incorrect because operating unit is also a generic term that may
encompass all the responsibility centers as centers of business operations.

30. In what type of center are managers usually evaluated on the basis of their fixed
costs and the contribution margin they provide to the company?
Chapter 8 Responsibility Accounting 73

A. Profit center. C. Investment center.


B. Cost center. D. Marketing center.
(rpcpa)

30. A
? The type of center managers that are usually evaluated on the basis of their fixed
costs and the contribution margin.
 When a manager is evaluated on the basis of his fixed costs and the contribution
margin he provides to the company, that manager is a profit center manager. A profit
center manager is accountable to the direct margin of a segment. (i.e., contribution
margin less direct fixed costs). Choice-letter “a” is the correct answer.
Choice-letter “b” is incorrect because a cost center manager is evaluated on the
amount of costs under his control. Choice-letter “c” is incorrect because an
investment manager is normally evaluated on the degree or amount of return on their
controllable investment. Choice-letter “d” is incorrect because a marketing center
manager is to be evaluated on the amount or rate of revenues his segment has
generated.

31. A center that incurs costs and expenses, generates revenue but does not have
control over idle funds used for investment purposes
A. Profit center. C. Cost center.
B. Investment center. D. Responsibility center. (rpcpa)

31. A
? A center that incurs costs and expenses, generates revenue but does not have
control over idle funds used for investment purposes.
 Choice-letter ”a”, profit center is correct. A profit center manager controls the
generation of revenues and the incurrence or use of costs and expenses and is to be
evaluated on the overall profitability of a segment in the short-term.
Choice-letter “b” is incorrect because an investment center manager is
responsible also in generating acceptable amount or rate of return on investment.
Choice-letter “c” is incorrect because a cost center manager is responsible only in
controlling costs. Choice-letter “d” is incorrect because a responsibility center
manager may be any of those managers mentioned where their duties are assigned
in an organization structured on responsibility centers.

32. Controllable revenue is included in a performance report for a


Profit center Cost center
A. No No
B. No Yes
C. Yes No
D. Yes Yes (aicpa)

32. C
? Whether controllable revenue is included in the performance report of profit center
and cost center.
Chapter 8 Responsibility Accounting 74

 Controllable revenue is within the sphere of authority of revenue center manager,


profit center manager, and investment center manager, but not of cost center
manager who only controls costs.

33. Which of the following types of responsibility centers has accountability for revenues?
A. Cost center and investment centers.
B. Profit centers and investment centers.
C. Cost centers and profit centers.
D. Expense and investment centers. (rpcpa)

33. B
? The responsibility center that has authority over revenues.
 In a decentralized organization, business units/segments are treated as responsibility
centers where a manager is authorized and also responsible and accountable on his
center’s performance. Responsibility centers are classified as: cost center, revenue,
profit center, and investment center. A cost center manager controls revenues only. A
profit center manager controls both revenues and costs. an investment manager
controls investments, revenues and costs.
The centers that have authority or control over revenues are the profit center and
investment center.

34. Profit centers


A. Have responsibility for controlling costs as well as capital.
B. Control and reports costs only.
C. Are the same as investment centers.
D. Measures income and relate that income to their invested capital. (rpcpa)

34. A
? A statement describing profit centers.
 Under the responsibility accounting system, organizational divisions, units and
segments are divided into responsibility centers which are classified as cost center,
revenue center, profit center, and investment center.
A profit center is managed by a person who is given the authority (power),
responsibility and accountability to control or influence the generation of revenue and
incurrence of costs (choice-letter “a” is correct).
Choice-letter “b” refers to costs center where a manager controls and influences
only the incurrence or non-incurrence of costs. Choice-letter “c” investment center,
gives the investment manger the authority and responsibility to control investment
and the return on investment (i.e., revenue, costs and expenses). Choice-letter “d”
refers to an evaluation technique used in measuring an investment manager’s
performance.

35. A major problem in comparing profitability measures among companies is the


A. Lack of general agreement over which profitability.
B. Differences in the size of the companies.
C. Differences in the accounting methods used by the companies.
D. Differences in the dividend policies of the companies. (cma)
Chapter 8 Responsibility Accounting 75

35. C
? A major problem in comparing profitability measures among companies.
 One of the major problems in comparing profitability measures is the differences in
the accounting methods among companies . This variability in accounting methods
results to impairment of comparability measures, hence, choice-letter “c” is incorrect.
Choice-letter “a” is incorrect because measures of profitability are widely
accepted among companies and analysts. Choice-letter “’b” is incorrect because
differences in the size of the companies may be remedied by vertical (ratio) analysis.
Choice-letter “d” is incorrect because dividend policy relates more to measures of
liquidity and potentials of company’s stock.

Questions 36 and 37 are based on the following information. A and B are


autonomous divisions of a corporation. They have no beginning or ending
inventories, and the number of units produced is equal to the number of units sold.
Following is financial information relating to the two divisions.
A B
Sales P150,000 P400,000
Other revenue 10,000 15,000
Direct materials 30,000 65,000
Direct labor 20,000 40,000
Variable factory overhead 5,000 15,000
Fixed factory overhead 25,000 55,000
Variable S & A expense 15,000 30,000
Fixed S & A expense 35,000 60,000
Central corporate expenses (allocated) 12,000 20,000

36. What is the total contribution to corporate profits generated by Division A before
allocation of central corporate expenses?
A. P18,000 C. P30,000
B. P20,000 D. P90,000 (cia)

36. C
? The total contribution to corporate profits generated by Division A before allocation of
central corporate expenses.
 The contribution to profit before allocation of central corporate expenses is called the
segment margin, calculated as follows:
Sales P150,000
Other revenue 10,000
Direct materials ( 30,000)
Direct labor ( 20,000)
Variable factory overhead ( 5,000)
Variable S & A expense ( 15,000)
Contribution margin 90,000
Fixed factory overhead (direct) ( 25,000)
Fixed S & A expense (direct) ( 35,000)
Segment margin P 30,000
37. What is the contribution margin of Division B?
A. P150,000 C. P235,000
Chapter 8 Responsibility Accounting 76

B. P205,000 D. P265,000 (cia)

37. D
? Contribution margin of Division B.
 Contribution margin is the difference between revenues and variable costs and
expenses, as follows:
Sales P400,000
Other revenue 15,000
Direct materials ( 65,000)
Direct labor ( 40,000)
Variable factory overhead ( 15,000)
Variable S & A expense ( 30,000)
Contribution margin P265,000

38. The segment margin of the Wire Division of Lerner Corporations should not include
A. Net sales of the Wire Division.
B. Fixed selling expenses of the Wire Division.
C. Variable selling expenses of the Wire Division.
D. The Wire Division’s fair share of the salary of Lerner Corporation’s president. (cma)

38. D
? The item not to be included in the segment margin.
 Segment margin is the difference between contribution margin and direct fixed costs
and expenses. The other names of segment margin are product margin, department
margin, and division margin. Allocated fixed expenses, which are indirect in nature,
are not included in the determination of segment margin. The Wire Division’s share
of the salary of the company president is an allocated cost and is not included in the
computation of the segment margin.
Choice-letters “a”, “b”, and “c” are all included in the calculation of segment
margin and are not the correct answers.

39. When using a contribution margin format for internal reporting purposes, the major
distinction between segment manager performance and segments performance is
A. Unallocated fixed cost.
B. Direct variable costs of producing the product.
C. Direct fixed cost controllable by the segments manager.
D. Direct fixed cost controllable by others. (cma)

39. D
? The major distinction between segment manager performance and segment
performance.
 Segment manager should be evaluated based on matters that he controls or
influences. The segment manager should be evaluated based on the controllable
margin and the segment performance should be evaluated based on segment
margin. The difference between the controllable margin and segment margin is the
direct controllable fixed costs and expenses, highlighted as follows:
Contribution margin Px
- Direct controllable fixed costs and expenses x
Chapter 8 Responsibility Accounting 77

Controllable margin x
- Direct non-controllable fixed costs and expenses x
Segment margin x
- Indirect fixed costs and expenses x
Operating income P x
Choice-letters “a” is incorrect because unallocated fixed costs is the indirect fixed
costs (i.e., common fixed costs) which is not included in the determination of segment
margin. Choice-letter “b” is incorrect because direct variable production costs are
common to both controllable margin and segment margin. Choice-letter “c” is
incorrect direct controllable fixed costs are also common to, and not a difference
between, controllable margin and segment margin.

40. Jun Iglesias is the manager of Profit Center #5. His unit reported the following for the period
just ended:
Contribution margin P350,000
Period expenses:
Manager’s salary P100,000
Depreciation expense 40,000
Allocated administrative costs 25,000 165,000
Profit center #5 income P185,000
Of the foregoing, in all likelihood, Mr. Iglesias controls
A. P165,000 C. P100,000
B. P185,000 D. P350,000 (rpcpa)

40. D
? The amount of controllable item to Mr. Iglesias.
 A cost, item or object is controllable to a manager if he can decide or influence the
incurrence or non-incurrence of such cost, item or object. Since Mr. Iglesias is a
profit center manager, he has control over revenues and variable costs and
expenses, and therefore, of the contribution margin. He does not control his salary,
depreciation expense (because it is normally under the control of an investment
manager) and allocated administrative costs (which is normally unavoidable). The
controllable value with respect to Mr. Iglesias, a profit center manager, is up to the
amount of the contribution margin.

41. The following is a summarized income statement of Carr Co.’s profit center No. 43
for March:
Contribution margin P70,000
Period expenses:
Manager’s salary P20,000
Facility depreciation 8,000
Corporate expense allocation 5,000 (33,000)
Profit center income P37,000
Which of the following amounts is most likely subject to the control of the profit
center’s manager?
A. P70,000 C. P37,000
B. P50,000 D. P33,000 (aicpa)
Chapter 8 Responsibility Accounting 78

41. A
? The amount subject to the control of a profit center manager.
 A profit center manager controls the generation of revenues and incurrence of costs
within the range of the power assigned to him. The costs controllable by the profit
center manager include the incremental revenues and the incremental costs and
expenses. Hence, the contribution margin, amounting to P70,000, is controllable as
far as the profit center manager is concerned.
The profit center manager is not in a position to control his salary. His salary is
controllable by his superiors. Facility depreciation is controllable by the investment
manager who is authorized to decide on matters of long-term assets acquisition, and
correspondingly of depreciation expense. Corporate expense allocation is
controllable by the corporate headquarter and not by an operating manager like a
profit center manager.

Investment centers
42. Decentralized firms can delegate authority and yet retain control and monitor
manager’s performance by structuring the organization into responsibility centers.
Which one of the following organizational segments is most likely an independent
business?
A. Revenue center. C. Cost center.
B. Profit center. D. Investment center.

42. D
? The most likely independent segment in an organization structured into responsibility
centers.
 Choice-letter “d” , investment center, is the most likely independent center in an
organization. An investment center manager is given the authority to decide on the
type of investment that business resources would be placed into. Correspondingly, it
has also the authority to manage revenues, costs, expenses, and profit.
Choice-letters “a”, “b”, and “c” are responsibility centers in the an organization
where there managers are of more limited authority compared with that of the
investment center. Revenue center managers have authority to decide revenue
matters only. Cost center managers have the authority to decide on costs matters
only. Profit center managers have authority to decide on revenues, costs, and profit
only.

43. A segments of an organization is investment center if it has


A. Authority to make decisions affecting the major determinants of profit, including
the power to choose its makers and sources of supply.
B. Authority to make decisions affecting the major determinants of profit, including
the power to choose its markets and sources of supply and significant control
over the amount of invested capital.
C. Authority to make decisions over the most significant costs of operation,
including the power to choose the sources of supply.
D. Responsibility for developing for markets for, and selling the output of, the
organization. (cma)
Chapter 8 Responsibility Accounting 79

43. B
? A true statement about an investment center.
 An investment center is headed by an investment center manager who has the
unique authority to decide on what business opportunity shall available funds be
ivnested into, including the basic authority to decide on factors affecting choice of
market, sources of supply, profit.
Choice-letter “a” is incorrect because it refers to profit center. Choice-letter “c” is
incorrect because it relates to cost center. Choice-letter “d” is incorrect because it
refers to revenue center.

44. Identify the following statements as true or false.


Statement 1-The main difference between a profit center and an investment center is
that the emphasis is on the rate of return in the investment center rather than an
absolute profit.
Statement 2-Marginal cost is the amount of cost increase caused by a unit increase
in the output of product.
Statement 1 Statement 2
A. False True
B. False False
C. True True
D. True False
(rpcpa)

44. C
? Identify the statements as either true or false.
 Statement 1 is a true statement, although awkwardly constructed. The emphasis in
evaluating an investment center manager is the return on investment. The emphasis
in evaluating a profit center manager is the amount of profit.
Statement 2 is also a true statement. Marginal cost is an increase in cost per unit
produced. The total increase in costs out of the total production is called as
incremental costs.

45. A segment of an organizations is referred to as a service center if it has


A. Responsibility for developing markets and selling the output of the organization.
B. Responsibility for combining the raw materials, direct labor, and other factors of
production into a final output.
C. Authority to make decisions affecting the major determinants of profit including
the power to choose its markets and sources of supply.
D. Authority to provide specialized support to other units within the organization. (cma)

45. D
? A description of a service center.
 Organizational centers may also be classified as producing centers or service
centers. A service center provides specialized support to producing centers.
Examples of service centers in a manufacturing firm are accounting department,
payroll department, maintenance department, canteen, health and sports clinics,
security department, information technology department, and similar specialized
departments. A producing center is directly involved in generating products and
Chapter 8 Responsibility Accounting 80

services, identifying and creating markets, and identifying sources of supply that are
directly identified with the efforts of an organization to generate revenues.
Choice-letters “a”, “b”, and ‘c” are incorrect because they refer to producing
centers.

46. A management decision may be beneficial for a given profit center but not for the
entire company. From the overall company viewpoint, the decision leads to
A. Suboptimization. C. Goal congruence.
B. Centralization. D. Maximization. (aicpa)

46. A
? The description for making a decision that gives benefit for a given profit center but
not for the entire company.
 Independent organizational segments should still be in one in advancing and
promoting the overall interest of the entire organizations, which refers to the overall
optimization model or goal congruence. There are several models of attaining this
objective. One mode is the concept that what is good for a independent part would
be good for the whole. This suggests consistency in benefits to both the part and the
whole. The other mode is the concept that what is good for the entire organization is
good for its individual parts. This may not be always true. Whenever there is a
conflict in deciding which interest (or benefit) should be prioritized, it is a corporate
principle to uphold the interest of the entire organization. However, when a manager
upholds the interest of his department over and above the entire organizational
interest, this is called suboptimization.
Choice-letter “b” is incorrect because centralization refers to the center or source
of power and not the effects of the using the power (or authority). Choice-letter “c” is
incorrect because goal congruence upholds the overall interest of the organization
and its subunits. Choice-letter “d” is incorrect because maximization relates to
optimizing limited resources to produce the most benefit that could be derived from it,
without regard to its effects to other organizational segments.

Investment center performance models


47. The invested capital employed turnover rate would include
A. Net income in the numerator. C. Sales in the numerator.
B. Net income in the denominator. D. Sales in the denominator. (aicpa)

47. C
? An item included in the invested capital-employed turnover.
 The invested capital-employed turnover, also called the assets turnover (or
investments turnover), is computed by dividing net sales over average assets. This
rate measures the effectiveness of management in utilizing assets to generate sales.
Choice-letters “a” and “b” are incorrect because net income is not used in the
assets turnover ratio. Choice-letter “d” is incorrect because sales should be in the
numerator and not in the denominator.

48. The invested capital-employed turnover rate would include


A. Invested capital in the denominator.
Chapter 8 Responsibility Accounting 81

B. Net income in the numerator.


C. Invested capital in the numerator.
D. Sales in the denominator. (rpcpa)

48. A
? The item included in the computation of invested capital employed turnover ratio.
 Also known as the “Assets Turnover Ratio”, invested-capital employed ratio is
computed by dividing net sales with amount of capital employed. Therefore, net sales
are the numerator and capital is the denominator. Hence, choice-letter “a” is correct.

49. Assuming that the net sales and net income remain the same, a company’s return on
investment would
A. Increase if the invested capital-employed turnover rate decreases.
B. Increase if the invested capital-employed turnover rate increases.
C. Increase if invested capital increases.
D. Decrease if invested capital decreases. (aicpa)

49. B
? The effect to return on investment assuming net sales and net income remain the
same.
 Return on investment (ROI) is operating income over average investments. An
expanded equation may also be used to compute ROI, that is, by multiplying return
on sales and assets turnover, as follows:
ROI = Return on sales x Assets turnover
= (Segment income / Net sales) x (Net sales / Average assets)

The invested capital-turnover rate is the same as the assets turnover. If the
return on sales or assets turnover increases, ROI is expected to increase. Hence,
choice-letter “b” is correct.
Choice-letter “a” is incorrect because a decrease in assets turnover decreases
ROI. Choice-letter “c” is incorrect because if invested capital increases, ROI
decreases. And choice-letter “d” is incorrect because if invested capital decreases,
ROI increases.

50. Identify whether the following statements are true or false.


Statement 1-Return on investment is equal to net income divided by capital turnover.
Statement 2-Return on investment is equal to capital investment divided by net
income.
Statement 3-In general, it is most desirable to use the same criteria in evaluating the
performance of the division manager and the performance of the division as an
investment by the corporation.
Statement 1 Statement 2 Statement 3
A. True True True
B. False False True
C. True True False
D. False False False (rpcpa)

50. D
Chapter 8 Responsibility Accounting 82

? Identify the statements as true or false.


 Statements 1 and 2 are false statements because return on investment is operating
income divided by the average investment. Statement 3 is also a false statement
because a division manager is evaluated based on controllable margin while the
division’s performance is measured based on the segment (or division) margin. The
evaluation base between the division manager’s performance and that of the division
itself is different.

Questions 51 and 52 are based on the following information. The following selected
data pertain to Jona Company’s Jaja Division for 2006:
Sales P1,000,000
Variable costs 600,000
Traceable fixed costs 100,000
Average invested capital 200,000
Imputed interest rate 15%
51. How much is the residual income?
A. P100,000 C. P300,000
B. P270,000 D. P330,000 (aicpa)

51. B
? The amount of residual income.
 Residual income is the difference between segment income and minimum income
(i.e., desired income, imputed income). It is a measure of an investment manager’s
performance based on absolute peso standard and is preferable than the ROI model.
The residual income is P270,000 computed s follows:
Operating income (P1,000,000 – P600,000 – P100,000) P300,000
- Minimum income (P200,000 x 15%) 30,000
Residual income P270,000

52. How much is the return on investment?


A. 75% C. 150%
B. 135% D. 200% (aicpa)

52. C
? The return on investment.
 Return on investment (ROI) is operating income divided by average investments.
This model of evaluating investment manager’s performance uses rate instead of
nominal peso value in the evaluation process and, thus, does not consider the life of
investment and the relative amount of investment. The calculation of the ROI is as
follows:
ROI = P300,000/P200,000 = 150%

53. Country Road Corporation has these selected data:


Units to be sold 25,000
Total cost of the units P 500,000
Fixed capital investment 1,00,000
Variable capital on sales 20%
Chapter 8 Responsibility Accounting 83

What should be the unit selling price to have a 20% return on investment?
A. P 28.00 C. P 30.00
B. P 29.17 D. P31.20 (rpcpa)

53. B
? The unit selling price to have a 20% return on investment.
 Unit sales price (USP) is the sum of unit cost (UC) and unit profit margin (UPM),
expressed as follows:
USP = UC + UPM
UC = P500,000/25,000 units = P20
The total capital (TC) is composed of fixed capital (FC) and variable capital (VC),
20% of sales:
If, Sales = 25,000USP
Then, VC = 20%(25,000USP) = 5,000USP
And, TC = FC + VC = P1,000,000 + 5,000USP
Therefore, UPM = 20%[(P1,000,000 + 5,000USP) / 25,000]
= 8 + .04USP
Finally, USP = UC + UPM
= P20 + (8 + 0.04USP)
= P29.17

54. Information concerning product X of Paradise Corporation for the year ended 2005 is
as follows:
Sales P 1,850,000
Margin 10%
Return on investment 20%
Minimum required rate of return on investment 15%
The residual income for product X is
A. P138,750 C. P 92,500
B. P 46,250 D. P185,000 (rpcpa)

54. B
? The residual income for product X.
 Residual income is the difference between segment income and minimum income.
The segment income is P185,000 (i.e., P1,850,000 x 10%). And, the investment
balance is P925,000 (i.e., P185,000/0.20). The residual income is P46,250,
computed as follows:
Segment income P185,000
- Minimum income (P925,000 x 15%) 138,750
Residual income P 46,250

55. You are evaluating the performance of Department X of Banawe Corporation for the
year 2006. You are given the following facts:
Sales during the year amounted to P880,000 at 12% margin.
Return on investment was 20%.
Minimum required rate of return was 15%.
Chapter 8 Responsibility Accounting 84

Based on the above information, the department’s residual income for the year 2005
was
A. P 26,400 C. P 79,200
B. P105,600 D. P 42,240 (rpcpa)

55. A
? The residual income in 2006.
 Residual income measures the nominal peso difference between segment income
and minimum income (or imputed income). The segement income is P105,600 (i.e.,
P880,000 x 12%), and the amount of investments is P528,000 (i.e., P105,600/0.20).
The residual income is as follows:
SEgement income P105,600
- Minimum income (P528,000 x 15%) 79,200
Residual income P 26,400

56. Fortress Products have a residual net income of P1.8 million. If the imputed interest
rate is 16%, compute the ROI
A. 5% C. 15%
B. 10% D. not listed (rpcpa)

56. D
? The ROI.
 Return on investment is operating income divided by the average investment.
Exhausting analysis on the data given, there is no way that the operating income and
the average investment could be determined. The correct letter-choice should be “d”.

57. Boomster, Inc. generated the following results for the period just ended:
Sales P1.0 million
Net income 0.1 million
Capital investment 0.5 million
To arrive at the return on investment, the following should be used.
A. ROI = (5/10) X (10/1) C. ROI = (5/10) X (1/10)
B. ROI = (10/5) X (10/1) D. ROI = (10/5) X (1/10) (rpcpa)

57. D
? Computation of ROI (return on investment)
 ROI, also referred to as return on assets (ROA), is a technique used to measure the
performance of an investment center and investment manager. In an expanded
equation, it is computed as follows:
ROI = Return on sales x Assets turnover
ROI = Income/Net Sales x Net sales/Investment
Applying the data given in the problem, we have:
ROI = P0.1 million/P1.0 million x P1.0 million/P0.5 million
or:
ROI = (1/10) x (10/5), choice-letter “d” is correct.
Chapter 8 Responsibility Accounting 85

58.Power Corporation has two divisions X and Y. Division X is evaluating a project that
will earn a rate of return which is more than the imputed interest charge for the
invested capital, but less than the division’s historical return on invested capital.
Division X is evaluating a project that will earn a rate of return which is less than the
imputed interest charge for the invested capital, but is more than the division’s
historical return on invested capital. If the corporate objective is to maximize residual
income, the division should decide as follows:
A. Y accept and X reject. C. Y reject and X accept.
B. Y reject and X reject. D. Y accept and X accept. (rpcpa)

58. C
? If the corporate objective is to maximize residual income, how should proposed
projects X and Y be decided upon.
 Residual income is the difference between operating income and minimum
(desired/imputed) income. If the residual income is positive, the project is to
accepted.
In deciding on whether to accept a proposed project using the residual income
method, the rate or return must be greater than the imputed rate and therefore
project X is acceptable, while project Y has a lower rater of return that imputed rate
and therefore should be rejected.
The division’s historical rate of return is irrelevant using the residual income
method. If the company uses the return on investment method in evaluating
divisional performance, the historical rate of return would be more important in the
analysis that the imputed rate and the answer would have been different.

59. Which one of the following statements pertaining to the return on investments (ROI)
as a performance measurement is incorrect?
A. When the average age of assets differs substantially across segments of a
business, the use of ROI may not be appropriate.
B. ROI relies on financial measures that are capable of being independently
verified, while other forms of performance measures are subject to manipulation.
C. The use of ROI may lead managers to reject capital investment projects that can
be justified by using discounted cash flow models.
D. The use of ROI can make it undesirable for a skillful manager to take on trouble-
shooting assignments such as those involving turning around unprofitable
divisions. (cma)

59. B
? An incorrect statement about ROI as a performance measure.
 ROI relies on financial models to evaluate past performances. These financial
models are assumed to be independently verifiable and are therefore reliable. Other
measures of financial performance such as residual income method, economic value-
added method, equity spread, total shareholders’ return, and market value-added are
also quantitatively and financially-based and are also independently verifiable
because the financial database used in determining the ROI is the same financial
database used in determining other performance measures. Choice-letter “b” is a
false statement and is the correct answer.
Chapter 8 Responsibility Accounting 86

Choice-letters “a”, “c”, and “d” are true statements regarding return on
investment. The use of ROI diminishes in value when asset lives vary. ROI is not a
discounted model of evaluating returns. The use of ROI may not be appropriate in
evaluating performance of a skillful manager who would be unwillingly transferred to
trouble shoot an unprofitable divisions.

60. A firm earning a profit can increase its return on investments by


A. Increasing sales revenue and operating expenses by the same peso amount.
B. Decreasing sales revenues and operating expenses by the same percentage.
C. Increasing investments and operating expenses by the same peso amount.
D. Increasing sales revenue and operating expenses by the same percentage. (cma)

60. D
? The ways by which ROI may be increased.
 ROI is determined using an expanded formula as follows:
ROI = Income/Net sales x Net sales/Investment
ROI = Return on sales x Assets turnover
Mathematically, ROI is increased by increasing operating income and decreasing
investments. Choice-letter “d” is correct because increasing sales and operating
expenses by the same percentage would result to an increase in net income because
of a greater increase in sales than that of the operating expenses.
Choice-letters “a” and “c” are incorrect because increasing or decreasing sales
and expenses by the same amount would not affect operating income and ROI.
Choice-letter “b” is incorrect because it will result to lower net income, and lower ROI.

61. Most firms use return on investments (ROI) to evaluate the performance of
investments center managers. If top management wishes division managers to use
all assets without regard to financing, the denominators in the ROI calculation will be.
A. Total assets employed. C. Working capital plus other assets.
B. Shareholder’s equity. D. Total assets available. (cma)

61. A
? The denominator to be used in calculating ROI.
 The management wishes to use all assets without regard to financing. The
denominator to be used in the computation of ROI should be all of the assets
employed or used in the operations. Assets that are available but are not used in the
operating activities should not be included in the evaluation.
Choice-letter “b” is incorrect because shareholders’ equity per se does not reflect
effectiveness in utilizing resources but is more reflective of the ability of the business
to increase wealth on the financing strategy of the firm. Choice-letter “c” is incorrect
because the phrase “plus other assets” signifies to include all assets even not used in
the operations. Choice-letter “d” is incorrect because total assets available does not
mean total assets employed.

62. Which one of the following items would most likely not be incorporated into
calculation of a division’s investment base when using the residual income approach
for performance measurement and evaluation?
A. Fixed assets employed in division operations.
Chapter 8 Responsibility Accounting 87

B. Land being held by the division as a site for a new plant.


C. Division inventories when division management exercises control over the
inventory levels.
D. Division accounts payable when division management exercises control over the
amount of short-term credit used. (cma)

62. B
? An item not likely to be included in the investment-base used in computing residual
income.
 The assets not to be included in the investment base are those not used or employed
in the normal operating cycle. An example is a land held for future plant site which is
presumed to have not been used in the current operations, idle, and should not be
included in the investment base.
Choice-letters “a” and “c: are incorrect because they are to be included in the
investment base. Choice-letter “d” is incorrect because it should be treated as a
deduction from working capital and is therefore effectively included in the investment
base.

63. Which denominator used in the return on investment (ROI) formula is criticized
because it combines the effects of operating decisions made at one organizational
levels with financing decisions made at another organizational level?
A. Total assets employed. C. Working capital plus other assets.
B. Shareholder’s equity. D. Total assets available. (cma)

63. B
? The denominator used in the ROI formula that is criticized because it relates
operating results with financing decisions.
 Operating results are essentially products of investing decisions. As a direct
consequence, investment in working capital produces sales. Long-term assets are
also used in the production cycle that have strategic but indirect effect in the
operating results. Because of this, operating results (i.e., operating income) should
be compared with investing decisions (e.g., investments in assets). On another
viewpoint, shareholders’ equity represents financing from owners and does not mirror
the effectiveness of investment decisions that have direct impact on operating
results. Choice-letter “b” is, therefore, correct.
Choice-letters “a”, “’c”, and “d” are incorrect because they do not relate to
financing decisions but rather to investing decisions.

64. The following information pertains to Bala Company for the year ended December 31,
2006:
Sales P600,000
Income 100,000
Capital Investment 400,000
Which of the following eqautions should be used to compute Bala’s return on
investment?
A. (4/6) x (6/1) = ROI C. (4/6) x (1/6) = ROI
B. (6/4) x (1/6) = ROI D. (6/4) x (6/1) = ROI (aicpa)
Chapter 8 Responsibility Accounting 88

64. B
? The equation that computes ROI.
 ROI is return on sales times assets turnover. Return on sales is operating income
divided by net sales. Assets turnover is net sales divided by investment. Applying
the formula, ROI shall be computed as 1/6 x 6/4.

65. Return on investment (ROI) is a very popular measure employed to evaluate the
performance of corporate segments because it incorporates all of the major
ingredients of profitability (revenues, cost, investment) into a single measure. Under
which one of the following combinations of actions regarding a segment’s revenues,
costs, and investment would a segment’s ROI always increase?

Revenues Costs Investment


A. Increase Decrease Increase
B. Decrease Decrease Decrease
C. Increase Increase Increase
D. Increase Decrease Decrease (cia)

65. D
? The behavior of revenues, costs, and investment that would increase ROI.
 Essentially, ROI is increased by increasing operating profit or decreasing investment.
To inch up operating income, sales should increase and costs should decrease.

66.The following information pertains to Quest Company’s Gold Division for the current
year:
Sales P311,000
Variable cost 250,000
Traceable fixed costs 50,000
Average invested capital 40,000
Imputed interest rate 10%
Quest’s return on investment was
A. 10.00% C. 27.50%
B. 13.33% D. 30.00% (aicpa)

66. C
? The return on investment.
 Return on investment (ROI) is operating income over average investments. The
operating income is P11,000 (i.e., P311,0000 – P250,000 – P50,000). Hence, the
ROI is 27.5% (i.e., P11,000/P40,000).

67. Listed below is selected financial information for Western Division of the Cybernet
Company for last year.
Account Amount (thousands)
Average working capital P 625
General and administrative expenses 75
Net sales 4,000
Average plant and equipment 1,775
Cost of goods sold 3,525
Chapter 8 Responsibility Accounting 89

If Cybernet treats the Western Division as an investments center for performance


measurement purposes, what is the before-tax return on investment (ROI) for last
year ?
A. 34.78% C. 19.79%
B. 22.54% D. 16.67% (cma)

67. D
? The return on investment.
 The operating income is P400,000 (i.e., P4,000,000 – P3,525,000 – P75,000), and
the total investment is P2,400,000 (i.e., P625,000 + P1,775,000). Therefore the ROI
is 16.67% (i.e., P400,000/P2,400,000).

68. Residual income is a performance evaluation that is used in conjunction with, or


instead of, return on investments (ROI). In many cases, residual income is preferred
to ROI because
A. Residual income is measure over time, while ROI represents the results for one
period.
B. Residual income concentrates on maximizing absolute pesos of income rather
than a percentage return as with ROI.
C. The imputed interest rate used in calculating residual income is more easily
derived than the target rate that is compared to the calculated ROI.
D. Average investments is employed with residual income while year-end
investments is employed with ROI. (cia)

68. B
? A reason for preferring residual income over ROI.
 Residual income is sometimes preferred over ROI because it measures absolute
peso excess of operating income over that of minimum income. The minimum
income is determined by multiplying investment by the desired rate of return, which
practically acts as the minimum rate of return. In essence, the residual income
considers both minimum rate of return and nominal peso analysis. Additionally, ROI
does not give relevant weight on the amounts of investment between alternatives
which may be of significance when comparing the impact and extent of decisions
made. Moreover, ROI disregards the life in years of assets employed
between/among alternatives.
Choice-letter “a” is incorrect because residual income and ROI are both
measures of operating cycle, over time, or in a given business period. Choice-letter
“c” is incorrect because the imputed interest rate used in residual income is
essentially the same as the target rate used in the ROI calculation. Normally, the
imputed rate is the cost of capital. Choice-letter “d” is incorrect because average
investments are used for both residual income and ROI methods.

69. Residual income is a better measure for performance evaluation of an investments


center manager than return on investments because
A. The problems associated with measuring the assets base are eliminated.
B. Desirable investments decisions will not be neglected by high-return divisions.
C. Only gross book value of assets needs to be calculated.
D. The arguments about the implicit cost of interest are eliminated. (cma)
Chapter 8 Responsibility Accounting 90

69. B
? A reason why residual income is a better performance measure than ROI.
 Residual income is the excess of operating income over minimum or desired income.
Residual uses nominal value of peso in determining the performance of an
investment. Based on this model, desirable investment decisions will not be
neglected by high-return divisions. Choice-letter “b” is correct.
Choice-letter “a” is incorrect because assets base is also used in residual income
specifically in the computation of minimum income. Choice-letter “c” is incorrect
because gross book value of assets is not the main basis of determining residual
income but the net amount of assets employed in the operating activities. Choice-
letter “d” is incorrect because the implicit (or imputed) rate of interest is not eliminated
but is used in computing residual income.

70. The imputed interest rate used in the residual income approach to performance
evaluation can best be described as the
A. Average lending rate for the year being evaluated.
B. Historical weighted-average cost of capital for the company.
C. Target return on investment set by the company’s management
D. Marginal after-tax cost of capital on new equity capital. (cma)

70. C
? A description of the imputed interest rate used in the residual income approach.
 Choice-letter “c” is correct. The imputed interest rate is based on target rate of return
set by the company’s management. The target rate of return serves as the minimum
rate of return desired by the management on a particular investment. The imputed
rate of return is normally the cost of capital.
Choice-letter “a” is incorrect because the average lending rate is only one of the
basis in determining the target rate. Choice-letter “b” is incorrect because the
historical weighted-average cost of capital does not reflect the new investment’s
target of rate of return. Choice-letter “d” is incorrect because the marginal after-tax
cost of capital on new equity capital is normally different from the weighted-average
cost of capital which is a better basis in determining the target return.

71.James Tan is the general manager of the Industrial Product Division, and his
performance is measured using the residual income method. Tan is reviewing the
following forecasted information for his division for next year:
Category Amount thousands)
Working capital P1,800
Revenue 30,000
Plant and equipment 17,200
If the imputed charge is charge is 15% and Tan wants to achieve a residual income
target of P2,000,000, what will costs have to be in order to achieve the target?
A. P 9,000,000 C. P25,150,000
B. P10,800,000 D. P25,690,000 (cma)

71. C
? The amount of cost to achieve a residual income target of P2,000,0000.
Chapter 8 Responsibility Accounting 91

 Costs and expenses shall be the difference between sales and operating income.
Since residual income is operating income less minimum income, therefore by
derivation, operating income is residual income plus minimum income. For purposes
of computing the minimum income, total assets are P19 million (i.e., P1.8 million +
P17.2 million). The computation for costs and expenses are as follows:
Revenues P30,000,000
- Operating income:
Residual income P2,000,000
+ Minimum income (P19 million x 15%) 2,850,000 4,850,000
Costs and expenses P25,150,000

72. RED Service Company is a computer service center. For the month of May, RED had
the following
Sales P450,000
Operating Income 25,000
Net profit after taxes 8,000
Total assets 500,000
Shareholder’s equity 200,000
Cost of capital 6%
Based on the above information, which one of the following statements is correct?
REB has a
A. Return on investment of 4%. C. Return on investments of 1.6%.
B. Residual income of P(5,000). D. Residual income of (22,000). (cma)

72. B
? The true statement that could be derived on the data given.
 The problem is asking for the computations of return on investment and residual
income, as follows:
* ROI = P25,000/P500,000 = 5%
* Residual income = ?
Operating income P25,000
- Minimum income (P500,000 x 6%) ( 30,000)
Residual income P(5,000)
The cost of capital serves as the target rate of return in determining minimum
income. Choice-letters “a”, “c”, and “d” are incorrect statements with regard to values
of return on investment and residual income.

73. Valdez Industries wants its division managers to concentrate on improving


profitability. The performance evaluation measures that are most likely to encourage
this behavior are
A. Dividends per share, return on equity, and times interest earned.
B. Turnover of operating assets, gross profit, margin, and return on equity.
C. Return an operating assets, the current ratio, and debt-to equity ratio.
D. Turnover of operating assets, dividends per share, and times interested earned. (cma)

73. B
Chapter 8 Responsibility Accounting 92

? The performance evaluation measures that are most likely to encourage focus on
improving profitability.
 Profitability is expressed in terms of accounting net income. Net income is the result
of generating sales and incurring costs and expenses. Sales are directly affected by
the amount of working capital or operating assets used in the normal business cycle.
As such, turnover on operating assets (e.g., Net sales/Operating assets) is classified
as a measure of profitability. Gross profit margin (e.g., gross profit/net sales) and
return on equity (e.g., net income/stockholders’ equity) are also measures of
profitability. Choice-letter “b” is, therefore, correct.
Choice-letter “a” is incorrect because dividend per share is a growth ratio and
times interest earned is a leverage ratio. Choice-letter “c” is incorrect because
current ratio is a liquidity ratio while debt-to-equity ratio is a leverage ratio. Choice-
letter “d” is incorrect because dividends per share is a growth ratio and times interest
earned is a leverage ratio; they are not profitability ratios.

74. Which of the following criteria would be most useful to sales department manager in
evaluating the performance of the manager’ customer service group?
A. The customer is always right.
B. Customers complain should be processed promptly.
C. Employees should maintain a positive attitude when dealing with customer
D. All customer inquiries should be answered within 7 days of receipt. (cia)

74. D
? The criterion most useful in evaluating the performance of the manager
 The criterion (e.g., standard) used in evaluating the performance of a customer
service group should be, among others, specific, measurable, understandable, and
realistic. A policy which states that customers’ inquiries should be answered within 7
days of receipt qualify as a standard in evaluating performance.
Choice-letters “a”, “b”, and “c” are not specific and measurable statements that
may be used in evaluating performances.

75. A major problem in comparing profitability measures among companies is the


A. Lack of general agreement over which profitability measure is best.
B. Differences in the size of the companies.
C. Differences in the accounting methods used by the companies.
D. Differences in the dividend policies of the companies. (cma)

75. C
? A major problem in comparing profitability measures among companies.
 Choice-letter “c” is correct. Comparing profitability measures among companies is
met with limitations. One of its major problems is the difference in the accounting
methods used by the companies. This matter results to substantial differences in
measuring operating results.
Choice-letter “a” is incorrect because there is a general agreement as to the
definition of profitability, courtesy of the generally accepted accounting principles.
Choice-letter “b” is incorrect because size of companies is not a major problem in
comparing profitability measures among companies by using the common-size
Chapter 8 Responsibility Accounting 93

analysis. Choice-letter “d” is also incorrect because differences in the dividend


policies of companies do not affect the manner by which profitability is measured.

Transfer pricing
76. The price that one division of a company charges another division for goods or
services provided is called the

A. Market price. C. Outlay price.


B. Transfer price. D. Distress price. (cia)

76. B
? The term that describes the price that one division of a company charges another
division for goods or services.
 Inasmuch as organizational segments are treated as independent business units,
they are allowed to transact business with external or internal parties in carrying their
tasks. It is most likely then that a division may be transacting with another division in
the same company. The price that is charged by the selling division to the buying
division is called a transfer price. A transfer price may be based on market price, cost
price, negotiated price, or dual pricing.
Choice-letter “a” is incorrect because market price is only one of the possible
transfer prices that may be used by the transacting divisions. Choice-letter “c” is
incorrect because outlay price may refer to cash payments which may not be the
price agreed upon by the transacting parties. Choice-letter “d” is incorrect because
distress price refers to the possible lowest price that may be offered by a selling party
simply to enter a market or dispose an asset for marginal amount of recovery.

77. An exchange of values between a business entity and parties outside thereof or a
transfer or use of assets or services within the business entity
A. Authorization. C. Transaction.
B. Accountability. D. Movements.

77. C
? An exchange of values between a business entity and parties outside thereof or a
transfer or use of assets or services within the business entity.
 Choice-letter “c”, transaction is an exchange of values between two parties, within the
business or with an outside party. Choice-letter “a” is incorrect because authorization
refers to approval or power granted to execute the transaction. Choice-letter “b” is
incorrect because accountability refers to answerability on the consequences relating
to the effects of transactions. Choice-letter “d” is incorrect because it does provide
relevant technical meaning in business.

78. Transfer pricing schemes can be based on


A. Market price. C. Negotiated price.
B. Cost –based price. D. All of the above. (rpcpa)

78. D
? The basis of transfer pricing.
Chapter 8 Responsibility Accounting 94

 A transfer price is the rate of money used to account for the transaction between two
divisions of the same economic entity. A transfer price may be the market price, a
cost-based price, a negotiated price, or a dual price. Choice-letter “d” is the correct
answer.

79. Gutierrez Corporation’s Department 1 produced component C that it is used by OZM


as a key part. Production and sales data for component C is as follows:

Selling price per unit P100


Variable cost per unit 60
Fixed cost per unit 24*
*based on 10,000 units capacity per annum
Gutierrez Corporation’s Department II is introducing a new product that will use
components C. An outside supplier has quoted Department I a price of P96 per unit.
This represents the usual P100 price less a quantity discount due to the large
number of Departments II’s requirements. The company has transfer price formula
of:
Transfer price = Variable cost per unit + Lost contribution margin per unit on outside sales
Department I has enough excess capacity to handle all of Department II’s needs. For
the overall interest of the company, Department I should
A. Sell to Department II at the same quoted price of P96 per unit.
B. Sell to Department II at minimum price of P60 per unit.
C. Not sell to Department II since it will lose P4 per unit.
D. Sell to Department II at P100 per unit. (rpcpa)

79. B
? The amount of unit transfer price (interdivisional transfer) should Department I sell its
product to Department II for the overall interest of the company.
 Transfer prices may be set at cost, market price, cost-plus price, or negotiated price.
A company’s optimizing transfer price is determined using the formula below:
Unit variable cost and expense Px
+ Unit lost contribution margin from an outside customers x
Unit Transfer Price Px
In determining a negotiated interdividisional transfer price, savings are deducted
lost income are added and profit from the interdivisioanl transfer may be allocated
between the selling and buying divisions.
In this problem, since Department I has excess capacity enough to
accommodate the orders of Department II, and therefore has opportunity costs, the
best optimizing transfer price for the overall interest of the company shall be the
amount of the unit variable cost, which is P60.00.

80. The first Division of Furrow Company produces part I that is used by OEN’s as a key
part in their products. Costs and sales data on Part I are as follows:
Selling price per unit P100
Variable cost per unit 60
Fixed cost per unit 24*
Chapter 8 Responsibility Accounting 95

* based on 40,000 units capacity per annum


Furrow Company’s second division is introducing a new product that will use Part 1.
An outside supplier has quoted Second division a price of P96 per unit. This
represents the usual P100 price less a quantity discount due to the large number of
Second Division’s requirement. If the second Division would buy 15,000 units of Part
1 from the First Division, the effect on the corporate profits would be:

A. Increase by P540,000. C. Increase by P210,000.


B. Increase by P1,500,000. D. Reduced by P60,000. (rpcpa))

80. A
? The effect on the overall income of the business if the First Division buys from the
Second Division.
 The outside market requires the Second Division to buy at P96 per unit. The First
Division has a variable cost of P60 per unit of producing the product. Since the
problem is silent, it is assumed that First Division has excess capacity to produce the
requirement of Second Division, and that First Division’s regular sales to other
customers would not be affected. In the overall perspective of the firm, it would be
better to produce the product at P60 per unit than buying it from an outside supplier
at P96 per unit and save P36 per unit in the process (i.e., P96 – P60). The total
company savings shall be P540,000 (15,000 units x P36).

81. The most fundamental responsibility center affected by the use of market-based
transfer prices is a (n)
A. Production center. C. Cost center.
B. Investment center. D. Profit center. (cma)

81. D
? The most fundamental responsibility center affected by the use of market-based
transfer prices.
 A market-based transfer price would affect the contribution margin of a selling
division which is basically classified as a profit center.
Choice-letter “a” is incorrect because production center, just like choice-letter “c”,
a cost center, is not affected by its profit performance but by its costs savings.
Choice-letter “b” is an inferior choice because investment center would not be as
significantly affected by the market-based transfer price as that of the profit center.

82. A limitation of transfer prices based on actual cost is that they


A. Charge inefficiencies to the department that is transferring the goods.
B. Can lead to suboptimal decisions for the company as a whole.
C. Must be adjusted by some markup.
D. Lack clarity and administrative convenience. (cia)

82. B
? A statement regarding limitation of transfer prices based on actual cost.
 A transfer price based on actual cost does not give motivation to the selling division
to reduce prices or produce profit. This model of transfer price may result to
Chapter 8 Responsibility Accounting 96

excessively accumulated costs that would be charged to the buying division and
would lead to long-term inefficiencies, thereby causing suboptimal decisions in favor
of a division but not for the company as a whole.
Choice-letter “a” is incorrect because the inefficiencies that may result using the
actual cost transfer pricing would be charged to the buying division and not the
selling division. Choice-letter “c” is incorrect because it contradicts the concept of
cost-based pricing. Choice-letter “d” is incorrect because cost-based pricing does
not lack clarity and even promotes administrative convenience.

83. Reyes Company has intracompany service transfers from Division Core, a cost
center, to Division Pro, a profit center. Under stable economic conditions, which of the
following transfer prices is likely to be most conducive to evaluating whether both
divisions have met their responsibilities?
A. Actual cost. C. Actual cost plus markup.
B. Standard variable cost. D. Negotiated price. (aicpa)

83. B
? The most conducive transfer price in evaluating whether the selling and buying
divisions have met their responsibilities under stable economic conditions.
 The selling division is a cost center and the buying division is a profit center. The
cost center is responsible in controlling costs only. Under stable economic
conditions, fixed costs are assumed to be constant, not controllable, and will not have
an impact on transfer prices to a profit center. Also under normal circumstances, an
independent selling division caters not only to interdivisional customers but to
external customers as well. Hence, only standard variable costs would be included in
the determining the transfer prices.
Choice-letter “a” is incorrect because actual cost as a basis of transfer pricing
does not promote long-term efficiencies and it also includes fixed costs which would
not be relevant in the intracompany transfer pricing under normal economic
conditions. Choice-letter “c” is incorrect because actual cost plus markup has two
imperfections – one, it is based on actual cost which does not protect the interest of
the buyer, and second, it has a markup which is not an interest of a selling cost
center. Choice-letter “d” is incorrect because negotiated price may result to a
variance that is not attributable to an interdivisional transaction and may simply
complicate the issue of meeting the basic responsibilities of the cost center and the
profit center

84. A large manufacturing company has several autonomous divisions that sell their
products in perfectly competitive external markets as well as internally to the other
divisions of the company. Top management expects each of its divisional managers
to take actions that will maximize the organization’s goal as well as their own goals.
Top management also promotes a sustained level of management effort of all of its
divisional managers. Under these circumstances, for products exchanged between
divisions, the transfer price that will generally lead to optimal decisions for the
manufacturing company would be a transfer price equal to the
A. Full cost of the product.
B. Full cost of the product plus a markup.
C. Variable cost of the product plus a markup.
Chapter 8 Responsibility Accounting 97

D. Market price of the product. (cia)

84. D
? The transfer price that generally leads to optimal decisions for a manufacturing
company.
 The market-based transfer pricing generally leads to optimal decisions for a
manufacturing company. This anchors on the premise that divisions are
independent companies and are to be evaluated separaately. Interdivisional
transactions are treated as arms-length transactions and should be shaped by
market forces. Market-based pricing therefore institutes competitiveness, optimal
sourcing, and market-driven efficiencies and productivity.
Choice-letters “a”, “b”, and “c” are incorrect because cost-based pricing
promotes long-term inefficiencies.

85. The Eastern division sells goods internally to the Western division of the same
company. The quoted external price in industry publications from a supplier near
Eastern is P200 per ton plus transportation. It cost P20 per ton to transport the goods
to Western. Eastern’s actual market cost per ton to buy the direct materials to make
transferred product is P100. Actual per-ton direct labor is P50. Other actual cost of
storage and handling are P40. The company president selects a P220 transfer price.
This is an example of
A. Market based transfer pricing. C. Negotiated transfer pricing.
B. Cost-based transfer pricing. D. Cost plus 20% transfer pricing. (cia)

85. A
? The transfer pricing model that arrives at a transfer price of P220.
 The transfer price of P220 is derived by adding the market price of P200 and the
transport price of P20.
Choice-letter “b” is incorrect because cost-based pricing would result to a transfer
price of P210 (i.e., P100 + P50 + P40 + P20). Choice-letter “c” is incorrect because
there is no terms of agreements arrived at a negotiation. Choice-letter “d” is incorrect
because it will result to P252 (i.e., P210 x 120%).

86. Which is of the following is the most significant disadvantage of a cost-based transfer
price.
A. Requires internally developed information.
B. Imposes market effects on company operations.
C. Requires externally developed information.
D. May not promote long-term efficiencies. (cia)

86. D
? A significant disadvantage of a cost-based transfer price.
 A cost-based transfer price allows the selling division to transfer all its costs incurred
in a production process regardless of its managerial effort, goal, and motivation This
may unduly increase costs and may not promote long-term efficiencies.
Choice-letters “a”, “’b”, and “c” are incorrect because they are not disadvantages
of cost-based transfer pricing.
Chapter 8 Responsibility Accounting 98

87. Which of the following is not true about international transfer prices for a multinational
firm.
A. Allows firms to attempt to minimize worldwide tax.
B. Allows firm to evaluate each division.
C. Provides each division with a profit-making orientation.
D. Allows firms to correctly price products in each country in which it operates. (cia)

87. D
? A statement that is not true about international transfer prices for a multinational firm
 International transfer pricing practiced by multinational firms carries with it the basic
advantages of transfer pricing such as establishing a standard of measuring
divisional performance, provides each division with a profit-orientation, delegating
authorities for quicker and better operating decisions, and a culture of friendly
competition. Additionally, a well-thought of international transfer pricing would allow
firms to minimize worldwide taxes by shifting costs and prices to countries that
provide favorable tax environment. Choice-letters “a”, “b”, and “c” are incorrect
because they are advantages of international transfer pricing.
Choice-letter “d” is the correct answer because an international transfer pricing
formula may bring benefits to country operations but is a disadvantage to another
country operations. Price setting differs from a country to another. Although transfer
pricing may have an impact on a country pricing, it is more correct to state however
that pricing in a country is decided by the market forces, traditions, economic
landscape, and other business variables. Hence, transfer pricing does not absolutely
allow firms to correctly price products in the country where they operate.

88. In a decentralized company in which decisions may buy goods from one another, the
transfer-pricing system should be designed primarily to
A. Aid in the appraisal and motivation of managerial performance.
B. Increase the consolidated value of inventory.
C. Allow division managers to buy from outsiders.
D. Minimize the degree of autonomous of division managers. (rpcpa)

88. A
? A primary reason for designing transfer pricing system.
 The concept of transfer pricing arises when at least two independent divisions within
a company (or conglomerate) enter into transactions with one another in which one of
them is a selling division and the other the buying division. In this case, the principle
of transfer pricing is overall (not divisional) profit optimization in relation to the “goal
congruence” objectives of the company. Along the way, the transfer pricing model
must aid in motivating and evaluating (or appraising) managerial performance so as
not to reduce the structural power of divisional autonomy and independence.
Choice-letter “b” is incorrect because the objective of the business is not to
increase inventory but to sell for profit. It is an antithesis to the profit objective of the
company.
Choice-letter “c” is incorrect because transfer pricing encourages managers to
consider transacting within the sphere of company’s operations in order to contain
any increase in the value of equity. Although divisional independence allows
Chapter 8 Responsibility Accounting 99

managers to transact with outsiders however, it does not conform to spillage in


wealth.
Choice-letter “d” is also incorrect because transfer pricing does not minimize the
degree of autonomy of division managers but encourages powerful collaboration.

89. An appropriate transfer price between two divisions of The Ocampo Company can be
determined from the following data.

Fabricating Division
Market price of subassembly P50
Variable cost of subassembly P20
Excess capacity (in units) 1,000
Assembling Division
Number of units needed 900
What is the natural bargaining range for the two divisions?
A. Between P20 and P50. C. Any amount less than P50.
B. Between P50 and P70. D. P50 is the only acceptable price. (cma)

89. A
? The natural bargaining range for the two divisions.
 The normal bargaining range between the buying and selling division is in between
the incremental costs of producing the product to the market price if bought from an
outside supplier. The buyer wanting to minimize cost would look at the incremental
costs of production as a possible transfer price. The seller wanting to maximize sales
would look at the market price as a transfer price. Choice-letter “a” is correct.
Choice-letter “b” is incorrect because P70 is already higher than the market price.
Choice-letter “c” is incorrect because any amount less than P50 would include an
amount less than P20 which is not acceptable to the selling division. Choice-letter “d”
does not speak of range, is incorrect because it is not only the P50 that is the
acceptable price but the price could be within the natural bargaining range of P20 to
P50.

90. Division A of a company is currently operating at 50% capacity. It produces a single


product and sells all its production to outside customers for P13 per unit. Variable
cost is P7 per unit, and fixed costs are P6 per unit at the current production level.
Division B, which currently purchases this product from an outside supplier for P12
per unit, would like to purchase the product from Division A. Division A will operate at
80% capacity to meet outside customers and Division B’s demand. What is the
minimum price that Division A should charge Division B for this product?
A. P 7.00 per unit. C. P12.00 per unit.
B. P10.40 per unit. D. P13.00 per unit. (cia)

90. A
? The minimum amount that Division A should charge Division B for a product.
 The minimum transfer price (MTP) if the selling division has an excess capacity shall
be:
MTP = Incremental costs + Opportunity costs of the excess capacity, if any
Chapter 8 Responsibility Accounting 100

The minimum transfer price to be charged by Division A to Division B should not


be lower than the incremental cost of producing the product, which in this case is
P7.00, the amount of the variable cost. There is no opportunity costs indicated as to
the alternative use of the excess capacity.

91. The Alpha Division of a company, which is operating at capacity, produces and sells
1,000 units of a certain electronic component in a perfectly competitive market.
Revenue and cost data are as follows:
Sales P50,000
Variable costs 34,000
Fixed costs 12,000
The minimum transfer price that should be charged to the Beta Division of the same
company for each component is
A. P12 C. P46
B. P34 D. P50 (cia)

91. D
? The minimum transfer price when a selling division is operating at capacity.
 The minimum transfer price (MTP) if the selling division has no excess capacity shall
be:
MTP = Unit sales price – Savings, if any
The unit sales price referred to here is the external market price or the unit sales
price offered by the selling division to its regular customers. The savings referred to
possible reduction in the cost of producing and distributing the product.
In this problem, the minimum transfer price shall be the market price of P50 (i.e.,
P50,000/1,000 units). Choice-letter “d” is correct.
Choice-letter “a” is incorrect because a P12 unit transfer price would only recover
fixed costs (e.g., P12.000/1,000 units). Choice-letter “b” is incorrect because a P34
unit transfer price would only recover the incremental variable cost (e.g.,
P34,000/1,000 units). This selling price may be possible if the selling division has
excess capacity enough to accommodate the order of buying division. sale. Choice-
letter “c” is incorrect because a P46 unit transfer price would only recover variable
costs and fixed costs but does not retain the profitability of the selling division.

Questions 92 through 94 are based on the following information. Cyberzone Inc.


has several divisions that operate as decentralized profit centers. Cyberzone’s
Entertainment Division manufactures video arcade equipment using the
products of two of Cyberzone’s other divisions. The Plastic Division
manufactures plastic components, one type that is made exclusively for the
Entertainment Division, while other less complex components are sold to outside
markets. The products of the Video Cards Division are sold in a competitive
market; however, one video card model is also used by the Entertainment
Division. The actual cost per unit used by the Entertainment Division are
presented in the next column.
Plastic Video
Components Cards
Chapter 8 Responsibility Accounting 101

Direct material P1.25 P2.40


Direct Labor 2.35 3.00
Variable Overhead 1.00 1.50
Fixed Overhead .40 2.25
Total Cost P5.00 P9.15
The Plastic Division sells its commercial products at full cost plus a 25% markup and
believes the proprietary plastic component made for the Entertainment Division would
sell for P6.25 per unit on the open market. The market price of the video card used
by the Entertainment Division is P10.98 per unit.

92. A per-unit transfer price from the Video Cards Division to the Entertainment Division
at full cost, P9.15, would
A. Allow evaluation of both divisions on a competitive basis.
B. Satisfy the Video Cards Division’s profit desire by allowing recovery of
opportunity cost.
C. Provide no profit incentive for the Video Cards Division to control or reduce cost.
D. Encourage the Entertainment Division to purchase video cards from an outside
source. (cma)

92. C
? The effect of using full cost as a transfer price from Video Cards Division to the
Entertainment Division.
 If the transfer price used is the full cost, the selling division, Video Cards Division,
would not be encouraged to produce and sell the products. A full-cost unit transfer
price does not provide profit incentive to the selling division and would not be
encouraged to control its costs. Choice-letter “c” is correct.
Choice-letter “a” is incorrect because a full-cost transfer price does not allow
evaluation of the buying and selling divisions on a competitive basis. Using this
transfer price model, the selling division has no profit to record and does not improve
its return on investment. Choice-letter “b” is incorrect because it does not satisfy the
profit desire of the selling division and that the opportunity cost (the profit from regular
sales that is foregone) would not also be recovered. Choice-letter “d” is incorrect
because a full-cost transfer price would initially encourage the buying division,
Entertainment Division, to purchase from the selling division within the enterprise.

93. Assume that the Entertainment Division is able to purchase a large quantity of video
cards from an outside source at P8.70 per unit. The Video Cards Division, having
excess capacity, agrees to lower its transfer price to P8.70 per unit. This action would
A. Optimize the profit goals of the Entertainment Division while subverting the profit
goals of Cyberzone Inc.
B. Allow evaluation of both divisions on the same basis.
C. Subvert the profit goals of the Video Cards Division while optimizing the profit
goals of the Entertainment Division.
D. Optimize the overall profit goals of Cyberzone Inc. (cma)

93. D
Chapter 8 Responsibility Accounting 102

? The effect if the Entertainment Division buys from the Video Cards Division at P8.70
per unit.
 The Video Cards Division has an excess capacity. This would mean that the video
Cards Division could lower its transfer price up to the incremental cost of producing
the units to be transferred. The incremental unit cost of Video Cards Division is
P6.90 (i.e., P2.40 + P3.00 + P1.50). Since the agreed unit transfer price is P8.70,
Video Cards Division could still make a divisional profit of P1.80 per unit (i.e., P8.70 –
P6.90). Most importantly, it also saves the entire company an amount of P1.80 per
unit. Hence, optimizing the overall profit goal of Cyberzone, Inc., choice-letter “d” is
the best answer.
Choice-letter “a” is incorrect because the agreed transfer price both optimizes the
profit goal of both the Entertainment Division and Cyberzone, Inc. Choice-letter “b” is
incorrect because the incremental profit of Video Cards Division is P1.80 while the
incremental savings of Entertainment Division is P2.28 (i.e., P10.98 – P8.70), hence,
the benefits to the divisions are not the same. Choice-letter “c” is incorrect because
the transfer price does not subvert the profit goal of Video Cards Division because it
would only use its excess capacity and would even maximize its profit along the way.

94. Assume that the Plastic Division has excess capacity and it has negotiated a transfer
price of P5.60 per plastic component with the Entertainment Division. This price will
A. Cause the Plastic Division to reduce the number of commercial plastic
components it manufactures.
B. Motivate both divisions as estimated profits are shared.
C. Encourage the Entertainment Division to seek an outside source for plastic
components.
D. Demotivate the Plastic Division causing mediocre performance. (cma)

94. B
? The effect of a transfer price of P5.60 per plastic component with the Entertainment
Division.
 Choice-letter “b” is correct because at a P5.60 unit transfer price, both divisions
would be benefited. The selling division, Plastic Division, would earn P1.00 per unit
(i.e., P5.60 – P4.60).. The total unit variable production costs of plastic is P4.60 (i.e.,
P1.25 + P2.35 + P1.00). Likewise, the buying division, Entertainment Division, would
also gain at a rate of P0.65 per unit (i.e., P6.25 – P5.60). Choice-letter “b” is correct.
Choice-letter “a” is incorrect because the P5.60 unit price would be less attractive
than the outside market sales price of P6.25. Hence, the regular market of plastic
products should not reduced. Choice-letter “c” is incorrect because the unit transfer
price would benefit Entertainment Division and would not encourage it to seek an
outside source for plastic components. Choice-letter “d” is incorrect because it would
not demotivate the Plastic Division because it would also benefit from the said
transfer price.

95. A company recently established a branch to sell its most popular fan. The company
purchases these fans and stores them in a warehouse. The fans are then shipped
from the warehouse to both the home office and the new branch, F.O.B. destination.
Home office management is responsible for setting the transfer price of the fans
Chapter 8 Responsibility Accounting 103

charged to the branch in a manner that will measure the long-run incremental cost of
supplying the fans to the branch on a continuing basis. Per unit costs for the fans are
P50.00 purchase price
P 2.50 shipping cost to warehouse
P 3.00 handling cost including P1.00 of allocated administrative overhead
P 3.50 shipping cost to branch paid by home office
P 1.25 shipping cost to home office

The minimum transfer price that home office should charge the branch to meet its
measurement objective is
A. P53.50 C. P58.00
B. P54.50 D. P59.00

95. D
? The minimum transfer price that home office should charge the branch to meet its
measurement objective.
 The branch should be charged only with those costs that are incurred in relation to
the purchase, storage, and delivery of fans to the branch location, as follows:
Purchase price P50.00
Shipping cost to warehouse 2.50
Handling costs 3.00
Shipping cost to branch 3.50
Price chargeable to branch P59.00

The handling cost includes the allocated administrative overhead because the
transfer price to be set should measure the long-run incremental cost of supplying the
fans to the branch on a continuing basis.

96. Macmod Company sells refrigeration components both in the Philippines and to a
subsidiary in Indonesia. One of the components, Part No. 333, has the variable
manufacturing cost of P30. The part can be sold domestically or shipped to the
Indonesian subsidiary for use in the manufacture of a residential subassembly.
Relevant data with regard to part no. 333 are shown below:
Domestic selling price P 65
Shipping charge to Indonesia 15
Cost of acquiring part no. 333 in Indonesia 75
Indonesian residential subassembly:
Sales price 170
Other manufacturing costs 55
Units shipped to Indonesia 150,000”
“If deemed preferable, these units could be sold in the Philippines.

The applicable tax rates are 40% in the Philippines and 70% in Indonesia. Macmod
Company will transfer part no. 333 to the Indonesian subsidiary at either variable
manufacturing cost or the domestic market price. On the basis of this information,
which one of the following strategies should be recommended to Macmod’s
management?
Chapter 8 Responsibility Accounting 104

A. Transfer 150,000 units at P30 and the Indonesian subsidiary pays the shipping
costs.
B. Transfer 150,000 units at P65 and the Indonesian subsidiary pays the shipping
costs.
C. Sells 150,000 units in the Philippines and the Indonesian subsidiary obtains part
no. 333 in Indonesia.
D. Transfer 150,000 units at P65 and the Philippine Company pays the shipping
costs.

96. C
? The strategy to be recommended to David’s management with regard to the options
given.
 This is a case of an inter-country, inter-divisional transfer. The best strategy for
Macmod Company is the one that maximizes its consolidated net income. The
consolidated net profit shall be the consolidated income before income tax less taxes
paid to the Philippines and Indonesian governments, as follows:
a b c d
Consolidated income before tax
[(P170 – P30 – P15 – P55) P70.00 P70.00 P70.00
[(P65 – P30) + (P170 – P75 – P55) P75.00
- Taxes paid
Philippines [(P30 – P30) 40%] -
[(P30 –P65 – P30) 40%] 14.00
[(P65 – P30 40%] 14.00
[(P65 – P30 – P15) 40%] 8.00
Indonesian [(P170 – P30 - P15 – P55) 70%] 49.00
[(P170 – P65 – P15 – P55) 70%] 24.50
[(P170 – P75 – P55) 70%] 28.00
[(P170 – P65 – P55)] 35.00
Consolidated net income P21.00 P31.50 P33.00 P27.00

Questions 97 and 98 are based on the following information: Magtalas Company


operates Division White and Division Red. Division White manufactures machine
tools on special order for outside market. Division Red manufactures metal lathes
which are sold to Division White as well as to outside market. Division White has job
order cost system and applies factory overhead at 75% of direct labor. As of June
30, 2006, Division White has only Job Order A in process and has been charged with
factory overhead of P25,200 and work-in-process account consisted of the following:
Balance, June 1 P 58,500
Direct materials, including transferred-in cost 170,000
Direct labor 125,000
Factory overhead 95,000
Transferred to finished goods (350,000)
Division Red has a process cost system and the cost to manufacture its product is
P12.00 per unit which is sold to Division White at 15% less than the selling price to
outside market. Sales price to outside market is P20.00.
Chapter 8 Responsibility Accounting 105

97. How much direct materials were charged to job Order A?


A. P 33,600 C. P 73,300
B. P 39,700 D. P 64,900 (rpcpa)

97. B
? The amount of direct materials charged to job order A.
 Job order A is the only job in process and it includes direct materials cost. To
compute for the amount of the direct materials charged in the work-in-process, let us
get first the ending balance of the work-in-process and deduct from it the conversion
costs charged in the same job in process, as follows:
In process balance, June 30, 2005
(P58,500 + P170,000 + P125,000 + P95,000 – P350,000) P98,500
Applied factory overhead ( 25,200)
Direct labor (P25,200/75%) ( 33,600)
Direct materials charged in work-in-process P39,700

98. How much is the transfer price for the machine lathes?
A. P 17.00 C. P 20.00
B. P 18.00 D. P 12.00 (rpcpa)

98. A
? The unit transfer price for the machine lathes.
 The unit transfer price for the machine lathers is 15% less than the market price of
P20 and therefore should be set at P17.00 (i.e., P20 x 85%).

Questions 99 through 103 are based on the following information. Oslo Company’s industrial
photo-finishing division, Rho, incurred the following cost and expenses in 2006:
Variable Fixed
Direct materials P200,000
Direct labor 150,000
Factory overhead 70,000 P42,000
General, selling and administrative 30,000 48,000
Totals P450,000 P90,000

During 2006, Rho produced 300,000 units of industrial photo-prints, which were sold
for P2.00 each. Oslo’s investment in Rho was P500,000 and P700,000 at January 1,
2006 and December 31, 2006, respectively. Oslo normally imputes interest on
investments at 15% of average invested capital.

99. For the year ended December 31, 2006, Rho’s return on average investment was
A. 15.0% C. 8.6%
B. 10.0% D. (5.0%) (aicpa)

99. B
? The rate or return on average investment.
 The average investment in 2006 is P600,000 [i.e., (P500,000 + P700,00) / 2]. The
operating income in 2006 is computed as follows:
Sales (300,000 units x P2) P600,000
Chapter 8 Responsibility Accounting 106

Variable costs ( 450,000)


Fixed costs ( 90,000)
Operating income P 60,000
Therefore, the return on investment should be 10% (i.e., P60,000/P600,000).

100. Assume that net operating income was P60,000 and that average invested capital
was P600,000. For the year ended December 31, 2006, Rho’s residual income (loss)
was
A. P150,000 C. P (45,000)
B. P 60,000 D. P (30,000) (aicpa)
100. D
? The amount of residual income.
 Residual income is the difference in operating income and minimum income, as
follows:
Operating income P60,000
- Minimum income (P600,000 x 15%) ( 90,000)
Residual income (P30,000)

101. How many industrial photo-print units did Rho have to sell in 2006 to break even?
A. 180,000 C. 90,000
B. 120,000 D. 60,000 (aicpa)
101. A
? The number of industrial photo-prints to breakeven.
 Breakeven point is fixed costs divided by the unit contribution margin. The total fixed
costs and expenses is P90,000. The unit contribution margin is P0.50, determined
as follows:
Unit sales price P2.00
- Unit variable cost (P450,000 / 300,000 units) ( 1.50)
Unit contribution margin P0.50
To breakeven, the company needs to sell 180,000 units (i.e., P90,000/P0.50)..

102.For the year ended December 31, 2006, Rho’s contribution margin was
A. P250,000 C. P150,000
B. P180,000 D. P 60,000 (aicpa)

102.C
? The amount of contribution margin.
 Contribution margin is the difference in net sales and variable costs and expenses.
The net amount of sales is P600,000 (i.e., 300,000 units x P2) while the variable
costs and expenses total P450,00. The contribution margin should be P150,000 (i.e.,
P600,000 – P450,000).

103.Assume the variable cost per unit was P1.50. Based on Rho’s financial data, and an
estimated 2007 production of 350,000 units of industrial photo-prints, Rho’s
estimated 2007 total cost and expenses will be
A. P525,000 C. P615,000
Chapter 8 Responsibility Accounting 107

B. P540,000 D. P630,000 (aicpa)

103.C
? The estimated total costs and expenses in 2006 if production is 350,000 units.
 The total costs and expenses are composed of variable costs and fixed costs, as
follows:
Variable costs and expenses (350,000 units x P1.50) P525,000
Fixed costs and expenses 90,000
Estimated total costs and expenses P615,000

Questions 104 through 116 are based on the following information.


Segment A Segment B Segment C Segment D
Net Income P5,000 -- -- P 90,000
Sales 60,000 P750,00 P135,000 1,800,000
Investment 24,000 500,000 45,000 --
Net income as % of sale -- -- -- --
Turnover of investment -- -- -- --
ROI -- -- 20% 7.5%
Minimum ROI-pesos -- -- -- P 120,000
Minimum ROI -% 20% 6% -- --
Residual income -- -0- P2,250 --

104.For Segment B, net income as a percentage of sales is


A. 8.00 % C. 4.00 %
B. 6.67 % D. 10.00 % (gleim)

104.C
? The net profit percentage for segment B.
 The net profit percentage equals net profit divided by net sales. The net profit is
P30,000 (i.e., P500,000 x 6%). Therefore, the net profit percentage is 4% (i.e.,
P30,000/P750,000).

105.For Segment C, net income as a percentage of sales is


A. 5.00 % C. 4.00 %
B. 6.67 % D. 20.00 %

105.B
? The net profit percentage for segment C.
 The net income is P9,000 (i.e., P45,000 x 20). Hence, the net profit ratio is 6.6.7%
(i.e., P9,000/P135,000).

106.For Segment C, the turnover of investment is


A. 3.0 C. 2.5
B. 1.5 D. 4.0

106.A
? The investment turnover for segment C.
Chapter 8 Responsibility Accounting 108

 Investment turnover is net sales divided by average investments. Hence, 3.0 for
segment C (i.e., P135,000/P45.000)

107.For Segment D, the turnover of investment is


A. 3.0 C. 2.5
B. 1.5 D. 4.0

107.B
? The investment turnover for segment D.
 Investment turnover may be also determined by dividing ROI over net profit ratio.
The profit ratio for segment D is 5% (i.e., P90,000/P180,000). Therefore, the
investment turnover is 150% (i.e., P7.5% / 5%).

108.For segment A, ROI is


A. 6.0 % C. 20.8 %
B. 2 0 % D. 7.5 %

108.C
? The ROI for segment A.
 Return on investment is net income over average investment. The ROI for segment
A is 20.8% (i.e., P5,000/P24,000.

109.For Segment B, ROI is


A. 6.0 % C. 20.0 %
B. 20.8 % D. 7.5 %

109.A
? The ROI for segment B.
 ROI is net income over average investments. Since the net income rate for segment
B is 4% (see no. 104), the net income is P30,000 (i.e., P4% x P750,000). And the
ROI for segment B is 6% (i.e., P30,000/P500,000).

110.For Segment A, the minimum peso ROI is


A. P30,000 C. P4,800
B. P6,750 D. P120,000

110.C
? The minimum peso ROI for segment A.
 The minimum peso ROI for segment A is P4,800 (i.e., P24,000 x 20%).

111.For Segment B. the minimum peso ROI is


A. P30,000 C. P 4,800
B. P 6,750 D. P120,000

111.A
? The minimum peso ROI for segment B.
 The minimum peso ROI for segment B is P30,000 (i.e., P500,000 x 6%).
Chapter 8 Responsibility Accounting 109

112.For Segment C, the minimum peso ROI is


A. P30,000 C. P 4,800
B. P 6,750 D. P120,000

112.B
? The minimum peso ROI for segment C.
 The minimum peso return may determined by deducting the residual income from the
segment income. The segment income is P9,000 (i.e., P45,000 x 20%). Therefore,
the minimum income is P6,750 (i.e., P9,000 – P2,250).

113.Assume that the minimum peso ROI is P6,750 for Segment C. The minimum
percentage of ROI is
A. 20 % C. 15 %
B. 6 % D. 10 %

113.C
? The minimum percentage of ROI for segment C.
 Minimum income percentage is minimum income over average investments. Hence,
the minimum income percentage for segment C is 15% (i.e., P6,750/P45,000).

114.In Segment D, the minimum percentage of ROI is


A. 20 % C. 15 %
B. 6 % D. 10 %

114.D
? The minimum percentage of ROI for segment D.
 The minimum profit rate is minimum profit in pesos divided average investments.
The average investment is P1,200,000 (i.e., P90,000/7.5%). Then the minimum
profit rate is 10% (i.e., P120,000/P1,200,000)

115.In Segment A, the residual income is


A. P 200 C. P(30,000)
B. P12,000 D. P 4,800

115.A
? The residual income for segment A.
 Residual income is the difference between net income and minimum peso income.
The net income is P5,000 and the minimum peso income is P4,800 (i.e., P24,000 x
20%). Hence, the residual income for segment D is P200 (i.e., P5,000 – P4,800).

116.In Segment D, the residual income is


A. P2,250 C. P(30,000)
B. P9,000 D. P 0

116.C
? The residual income for segment D.
 The residual income for segment D is P(30,000) which is the difference between the
net income of P90,000 and minimum peso income of P120,000.
Chapter 8 Responsibility Accounting 110

 done 

Вам также может понравиться